Exam 5 (lvl3)

Réussis tes devoirs et examens dès maintenant avec Quizwiz!

A client is concerned because she has been told her blood type and her baby's are incompatible. What is the nurse's best response? 1. "This is called ABO incompatibility. It is somewhat common but rarely causes significant hemolysis." 2. "This is a serious condition, and additional blood studies are currently in process to determine whether you need a medication to prevent it from occurring with a future pregnancy." 3. "This is a condition caused by a blood incompatibility between you and your husband, but does not affect the baby." 4. "This type of condition is very common, and the baby can receive a medication to prevent jaundice from occurring."

1 Explanation: 1. When blood types, not Rh, are incompatible, it is called ABO incompatibility. The incompatibility occurs as a result of the maternal antibodies present in her serum and interaction between the antigen sites on the fetal RBCs.

) A client with diabetes is receiving preconception counseling. The nurse will emphasize that during the first trimester, the woman should be prepared for which of the following? 1. The need for less insulin than she normally uses 2. Blood testing for anemia 3. Assessment for respiratory complications 4. Assessment for contagious conditions

1 Explanation: 1. Women with diabetes often require less insulin during the first trimester.

The nurse educator is presenting a class on the different kinds of miscarriages. Miscarriages, or spontaneous abortions, are classified clinically into which of the following different categories? Note: Credit will be given only if all correct and no incorrect choices are selected. Select all that apply. 1. Threatened abortion 2. Incomplete abortion 3. Complete abortion 4. Missed abortion 5. Acute abortion

1, 2, 3, 4 Explanation: 1. Unexplained cramping, bleeding, or backache indicates the fetus might be in jeopardy. This is a threatened abortion. 2. In an incomplete abortion, parts of the products of conception are retained, most often the placenta. 3. In a complete abortion, all the products of conception are expelled. The uterus is contracted and the cervical os may be closed. 4. In a missed abortion, the fetus dies in utero but is not expelled.

) The nurse is presenting a class on the pathophysiology of the different abortions. Some of the causes are which of the following? Note: Credit will be given only if all correct choices and no incorrect choices are selected. Select all that apply. 1. Chromosomal abnormalities 2. Insufficient or excessive hormonal levels 3. Sexual intercourse in the first trimester 4. Infections in the first trimester 5. Cervical insufficiency

1, 2, 4, 5 Explanation: 1. Chromosomal defects are generally seen as spontaneous abortions during weeks 4 to 8. 2. Insufficient or excessive hormonal levels usually will result in spontaneous abortion by 10 weeks' gestation. 4. Infectious and environmental factors may also be seen in first trimester pregnancy loss. 5. In late spontaneous abortion, the cause is usually a maternal factor, for example, cervical insufficiency or maternal disease, and fetal death may not precede the onset of abortion.

) Benefits of skin-to-skin care as a developmental intervention include which of the following? Note: Credit will be given only if all correct choices and no incorrect choices are selected. Select all that apply. 1. Routine discharge 2. Stabilization of vital signs 3. Increased periods of awake-alert state 4. Decline in the episodes of apnea and bradycardia 5. Increased growth parameters

2, 4, 5 Explanation: 2. Stabilization of vital signs is a benefit of skin-to-skin care as a developmental intervention. 4. Decline in the episodes of apnea and bradycardia is a benefit of skin-to-skin care as a developmental intervention. 5. Increased growth parameters are a benefit of skin-to-skin care as a developmental intervention.

The community nurse is working with a client at 32 weeks' gestation who has been diagnosed with preeclampsia. Which statement by the client would indicate that additional information is needed? 1. "I should call the doctor if I develop a headache or blurred vision." 2. "Lying on my left side as much as possible is good for the baby." 3. "My urine could become darker and smaller in amount each day." 4. "Pain in the top of my abdomen is a sign my condition is worsening."

3 Explanation: 3. Oliguria is a complication of preeclampsia. Specific gravity of urine readings over 1.040 correlate with oliguria and proteinuria and should be reported to the physician.

The nurse is admitting a client to the labor and delivery unit. Which aspect of the client's history requires notifying the physician? 1. Blood pressure 120/88 2. Father a carrier of sickle-cell trait 3. Dark red vaginal bleeding 4. History of domestic abuse

3. Third-trimester bleeding is caused by either placenta previa or abruptio placentae. Dark red bleeding usually indicates abruptio placentae, which is life-threatening to both mother and fetus.

) The client has just been diagnosed as diabetic. The nurse knows teaching was effective when the client makes which statement? 1. "Ketones in my urine mean that my body is using the glucose appropriately." 2. "I should be urinating frequently and in large amounts to get rid of the extra sugar." 3. "My pancreas is making enough insulin, but my body isn't using it correctly." 4. "I might be hungry frequently because the sugar isn't getting into the tissues the way it should."

4 Explanation: 4. The client who understands the disease process is aware that if the body is not getting the glucose it needs, the message of hunger will be sent to the brain.

Which nursing intervention is appropriate in the management of the preterm infant with hypothermia? Note: Credit will be given only if all correct choices and no incorrect choices are selected. Select all that apply. 1. Warm the baby rapidly to reverse the hypothermia. 2. Monitor skin temperature every 2 hours to determine whether the infant's temperature is increasing. 3. Keep IV fluids at room temperature. 4. Initiate efforts to maintain the newborn in a neutral thermal environment. 5. Warm the baby slowly to reverse hypothermia and reach a neutral thermal environment.

4, 5 Explanation: 4. The nurse should initiate efforts to block heat loss by evaporation, radiation, convection, and conduction. 5. The infant should be warmed slowly to prevent hypotension and apnea.

A primary herpes simplex infection in the first trimester can increase the risk of which of the following? 1. Spontaneous abortion 2. Preterm labor 3. Intrauterine growth restriction 4. Neonatal infection

: 1 Explanation: 1. A primary herpes simplex infection can increase the risk of spontaneous abortion when infection occurs in the first trimester.

A nurse is caring for a newborn on a ventilator who has respiratory distress syndrome (RDS). The nurse informs the parents that the newborn is improving. Which data support the nurse's assessment? 1. Decreased urine output 2. Pulmonary vascular resistance increases 3. Increased PCO2 4. Increased urination

: 4 Explanation: 4. In babies with respiratory distress syndrome (RDS) who are on ventilators, increased urination/diuresis may be an early clue that the baby's condition is improving.

A 3-month-old baby who was born at 25 weeks has been exposed to prolonged oxygen therapy. Due to oxygen therapy, the nurse explains to the parents, their infant is at a greater risk for which of the following? 1. Visual impairment 2. Hyperthermia 3. Central cyanosis 4. Sensitive gag reflex

Answer: 1 Explanation: 1. Extremely premature newborns are particularly susceptible to injury of the delicate capillaries of the retina causing characteristic retinal changes known as retinopathy of prematurity (ROP). Judicious use of supplemental oxygen therapy in the premature infant has become the norm.

The neonatal special care unit nurse is overseeing the care provided by a nurse new to the unit. Which action requires immediate intervention? 1. The new nurse holds the infant after giving a gavage feeding. 2. The new nurse provides skin-to-skin care. 3. The new nurse provides care when the baby is awake. 4. The new nurse gives the feeding with room-temperature formula.

Answer: 4 Explanation: 4. Preterm babies have little subcutaneous fat, and do not maintain their body temperature well. Formula should be warmed prior to feedings to help the baby maintain its temperature.

Which term refers to a newborn born before completion of week 37 of gestation, regardless of birth weight? a. Postterm b. Preterm c. Low birth weight d. Small for gestational age

B A preterm newborn is any child born before 37 weeks of gestation, regardless of birth weight. A postterm or postmature newborn is any child born after 42 weeks of gestational age, regardless of birth weight. A low birth weight newborn is a child whose birth weight is less than 2500 g, regardless of gestational age. A small-for-gestational-age (or small-for-date) newborn is any child whose rate of intrauterine growth was slowed and whose birth weight falls below the 10th percentile on intrauterine growth curves.

Which is the primary treatment for hypoglycemia in newborns with feeding intolerance? a. Oral glucose feedings b. Intravenous (IV) infusion of glucose c. Short-term insulin therapy d. Feedings (formula or breast milk) at least every 2 hours

B IV infusions of glucose are indicated when the glucose level is very low and when feedings are not tolerated. Early feedings in the normoglycemic newborn are preventive. When the newborn is unable to tolerate feedings or the blood glucose level has become extremely low, then IV infusions are indicated. Insulin administration will further depress the blood glucose level. Feedings can be preventive. The child may not be able to tolerate this frequency.

A healthy, stable, preterm newborn will soon be discharged. The nurse should recommend which position for sleep? a. Prone b. Supine c. Side lying d. Position of comfort

B The American Academy of Pediatrics recommends that healthy newborns be placed to sleep in a supine position. Other positions are associated with sudden infant death syndrome. The prone position can be used for supervised play.

Which is the most appropriate nursing intervention for the newborn who is jittery and twitching and has a high-pitched cry? a. Monitor blood pressure closely. b. Obtain urine sample to detect glycosuria. c. Obtain serum glucose and serum calcium levels. d. Administer oral glucose or, if newborn refuses to suck, IV dextrose.

C These are signs and symptoms of hypocalcemia and hypoglycemia. A blood test is useful to determine the treatment. Laboratory analysis for calcium and blood glucose should be the priority intervention. Monitoring vital signs is important, but recognition of the possible hypocalcemia and hypoglycemia is imperative. A finding of glycosuria would not facilitate the diagnosis of hypoglycemia. A determination must be made between the hypocalcemia and hypoglycemia before treatment can be initiated.

Which is an important nursing action related to the use of tape and/or adhesives on preterm newborns? a. Avoid using tape and adhesives until skin is more mature. b. Use solvents to remove tape and adhesives instead of pulling on skin. c. Remove adhesives with warm water or mineral oil. d. Use scissors carefully to remove tape instead of pulling tape off.

C Warm water, mineral oil, or petrolatum can be used to facilitate the removal of adhesive. In the preterm newborn, often it is impossible to avoid using adhesives and tape. The smallest amount of adhesive necessary should be used. Solvents should be avoided because they tend to dry and burn the delicate skin. Scissors should not be used to remove dressings or tape from the extremities of very small and immature newborns because it is easy to snip off tiny extremities or nick loosely attached skin.

Which refers to a newborn whose rate of intrauterine growth was slowed and whose birth weight falls below the 10th percentile on intrauterine growth charts? a. Postterm b. Postmature c. Low birth weight d. Small for gestational age

D A small-for-gestational-age (or small-for-date) newborn is any child whose rate of intrauterine growth was slowed and whose birth weight falls below the 10th percentile on intrauterine growth curves. A postterm or postmature newborn is any child born after 42 weeks of gestational age, regardless of birth weight. A low birth weight newborn is a child whose birth weight is less than 2500 g, regardless of gestational age.

The nurse is caring for a high-risk newborn with an umbilical catheter in a radiant warmer. The nurse notes blanching of the feet. Which is the most appropriate nursing action? a. Elevate feet 15 degrees. b. Place socks on newborn. c. Wrap feet loosely in prewarmed blanket. d. Report findings immediately to the practitioner.

D Blanching of the feet, in a newborn with an umbilical catheter, is an indication of vasospasm. Vasoconstriction of the peripheral vessels, triggered by the vasospasm, can seriously impair circulation. It is an emergency situation and must be reported immediately.

A preterm newborn of 36 weeks of gestation is admitted to the NICU. Approximately 2 hours after birth, the newborn begins having difficulty breathing, with grunting, tachypnea, and nasal flaring. Which is important for the nurse to recognize? a. This is a normal finding. b. This is not significant unless cyanosis is present. c. Improvement should occur within 24 hours. d. Further evaluation is needed.

D Difficulty breathing, with grunting, tachypnea, and nasal flaring are clinical manifestations of respiratory distress syndrome and require further evaluation. This is not a normal finding and requires further evaluation. Cyanosis may be present, but these are significant findings indicative of respiratory distress without cyanosis. The child's condition will most likely worsen for approximately 48 hours without intervention. Improvement may begin at 72 hours.

What causes meconium aspiration syndrome? a. Hypoglycemia b. Carbon dioxide retention c. Bowel obstruction with meconium d. Aspiration of meconium in utero or at birth

D Meconium aspiration syndrome is caused by the aspiration of amniotic fluid containing meconium into the fetal or newborn trachea in utero or at first breath. Hypoglycemia and carbon dioxide retention are not related to meconium aspiration. Bowel obstruction with meconium may be an indication of cystic fibrosis or Hirschsprung disease, not meconium aspiration.

Which are clinical manifestations of the postterm newborn? (Select all that apply.) a. Excessive lanugo b. Increased subcutaneous fat c. Absence of scalp hair d. Parchment-like skin e. Minimal vernix caseosa f. Long fingernails

D, E, F In postterm newborns, the skin is often cracked, parchment-like, and desquamating; there is little to no vernix caseosa; and fingernails are long. Lanugo is usually absent in postterm newborns. Subcutaneous fat is usually depleted, giving the child a thin, elongated appearance. Scalp hair is usually abundant.

) The nurse is caring for several pregnant clients. Which client should the nurse anticipate is most likely to have a newborn at risk for mortality or morbidity? 1. 37-year-old, with a history of multiple births and preterm deliveries who works in a chemical factory 2. 23-year-old of low socioeconomic status, unmarried 3. 16-year-old who began prenatal care at 30 weeks 4. 28-year-old with a history of gestational diabetes

Explanation: 1. This client is at greatest risk because she has multiple risk factors: age over 35, high parity, history of preterm birth, and exposure to chemicals that might be toxic.

A woman is hospitalized with severe preeclampsia. The nurse is meal-planning with the client and encourages a diet that is high in what? 1. Sodium 2. Carbohydrates 3. Protein 4. Fruits

: 3 Explanation: 3. The client who experiences preeclampsia is losing protein.

What is the most significant cause of neonatal morbidity and mortality? 1. Amenorrhea 2. Posttraumatic stress disorder 3. Prematurity 4. Endometriosis

3 Explanation: 3. The most significant cause of neonatal morbidity and mortality is prematurity and its associated complications such as respiratory distress syndrome, necrotizing enterocolitis, and intraventricular hemorrhage.

) The client with blood type A, Rh-negative, delivered yesterday. Her infant is blood type AB, Rh-positive. Which statement indicates that teaching has been effective? 1. "I need to get RhoGAM so I don't have problems with my next pregnancy." 2. "Because my baby is Rh-positive, I don't need RhoGAM." 3. "If my baby had the same blood type I do, it might cause complications." 4. "Before my next pregnancy, I will need to have a RhoGAM shot."

1 Explanation: 1. Rh-negative mothers who give birth to Rh-positive infants should receive Rh immune globulin (RhoGAM) to prevent alloimmunization.

What is the most significant maternal risk factor for preterm birth? 1. Previous preterm birth 2. Smoking 3. Stress 4. Substance abuse

1 Explanation: 1. The most significant maternal risk factor for preterm birth is a previous preterm birth.

The nurse caring for a postterm newborn would not perform what intervention? 1. Providing warmth 2. Frequently monitoring blood glucose 3. Observing respiratory status 4. Restricting breastfeeding

4 Explanation: 4. Breastfeeding is an appropriate means of feeding for the postterm newborn.

A woman at 7 weeks' gestation is diagnosed with hyperemesis gravidarum. Which nursing diagnosis would receive priority? 1. Fluid Volume: Deficient 2. Cardiac Output, Decreased 3. Injury, Risk for 4. Nutrition, Imbalanced: Less than Body Requirements

1 Explanation: 1. The newly admitted client with hyperemesis gravidarum has been experiencing excessive vomiting, and is in a fluid volume-deficit state.

The nurse prepares to admit to the nursery a newborn whose mother had meconium-stained amniotic fluid. The nurse knows this newborn might require which of the following? 1. Initial resuscitation 2. Vigorous stimulation at birth 3. Phototherapy immediately 4. An initial feeding of iron-enriched formula

1 Explanation: 1. The presence of meconium in the amniotic fluid indicates that the fetus may be suffering from asphyxia. Meconium-stained newborns or newborns who have aspirated particulate meconium often have respiratory depression at birth and require resuscitation to establish adequate respiratory effort.

A woman is 16 weeks pregnant. She has had cramping, backache, and mild bleeding for the past 3 days. Her physician determines that her cervix is dilated to 2 centimeters, with 10% effacement, but membranes are still intact. She is crying, and says to the nurse, "Is my baby going to be okay?" In addition to acknowledging the client's fear, what should the nurse also say? 1. "Your baby will be fine. We'll start IV, and get this stopped in no time at all." 2. "Your cervix is beginning to dilate. That is a serious sign. We will continue to monitor you and the baby for now." 3. "You are going to miscarry. But you should be relieved because most miscarriages are the result of abnormalities in the fetus." 4. "I really can't say. However, when your physician comes, I'll ask her to talk to you about it."

2 Explanation: 2. If bleeding persists and abortion is imminent or incomplete, the woman may be hospitalized, IV therapy or blood transfusions may be started to replace fluid, and dilation and curettage (D&C) or suction evacuation is performed to remove the remainder of the products of conception.

A client is being admitted to the labor area with the diagnosis of eclampsia. Which actions by the nurse are appropriate at this time? Note: Credit will be given only if all correct choices and no incorrect choices are selected. Select all that apply. 1. Tape a tongue blade to the head of the bed. 2. Pad the side rails. 3. Have the woman sit up. 4. Provide the client with grief counseling. 5. The airway should be maintained and oxygen administered.

2, 5 Explanation: 2. Side rails should be up and padded. 5. Suctioning may be necessary to keep the airway clear.

A client is admitted to the labor and delivery unit with a history of ruptured membranes for 2 hours. This is her sixth delivery; she is 40 years old, and smells of alcohol and cigarettes. What is this client at risk for? 1. Gestational diabetes 2. Placenta previa 3. Abruptio placentae 4. Placenta accreta

3 Explanation: 3. Abruptio placentae is more frequent in pregnancies complicated by smoking, premature rupture of membranes, multiple gestation, advanced maternal age, cocaine use, chorioamnionitis, and hypertension.

The nurse is caring for a client at 30 weeks' gestation who is experiencing preterm premature rupture of membranes (PPROM). Which statement indicates that the client needs additional teaching? 1. "If I were having a singleton pregnancy instead of twins, my membranes would probably not have ruptured." 2. "If I develop a urinary tract infection in my next pregnancy, I might rupture membranes early again." 3. "If I want to become pregnant again, I will have to plan on being on bed rest for the whole pregnancy." 4. "If I have aminocentesis, I might rupture the membranes again."

3 Explanation: 3. There is no evidence that bed rest in a subsequent pregnancy decreases the risk for PPROM.

The nurse is working with parents who have just experienced the birth of their first child at 34 weeks. Which statements by the parents indicate that additional teaching is needed? Note: Credit will be given only if all correct choices and no incorrect choices are selected. Select all that apply. 1. "Our baby will be in an incubator to keep him warm." 2. "Breathing might be harder for our baby because he is early." 3. "The growth of our baby will be faster than if he were term." 4. "Tube feedings will be required because his stomach is small." 5. "Because he came early, he will not produce urine for 2 days."

3, 4, 5 Explanation: 3. Preterm infants grow more slowly than do term infants because of difficulty in meeting high caloric and fluid needs for growth due to small gastric capacity. 4. Although tube feedings might be required, it would be because preterm babies have a marked danger of aspiration and its associated complications due to the infant's poorly developed gag reflex, incompetent esophageal cardiac sphincter, and inadequate suck/swallow/breathe reflex. 5. Although preterm babies have diminished kidney function due to incomplete development of the glomeruli, they can produce urine. Preterm infants usually have some urine output during the first 24 hours of life.

A 38-week newborn is found to be small for gestational age (SGA). Which nursing intervention should be included in the care of this newborn? 1. Monitor for feeding difficulties. 2. Assess for facial paralysis. 3. Monitor for signs of hyperglycemia. 4. Maintain a warm environment.

4 Explanation: 4. Hypothermia is a common complication in the SGA newborn; therefore, the newborn's environment must remain warm, to decrease heat loss.

A clinic nurse is planning when to administer Rh immune globulin (RhoGAM) to an Rh-negative pregnant client. When should the first dose of RhoGAM be administered? 1. After the birth of the infant 2. 1 month postpartum 3. During labor 4. At 28 weeks' gestation

4 Explanation: 4. When the woman is Rh negative and not sensitized and the father is Rh positive or unknown, Rh immune globulin is given prophylactically at 28 weeks' gestation.

A 20-year-old woman is at 28 weeks' gestation. Her prenatal history reveals past drug abuse, and urine screening indicates that she has recently used heroin. The nurse should recognize that the woman is at increased risk for which condition? 1. Erythroblastosis fetalis 2. Diabetes mellitus 3. Abruptio placentae 4. Pregnancy-induced hypertension

4 Explanation: 4. Women who use heroin are at risk for poor nutrition, anemia, and pregnancy-induced hypertension (or preeclampsia-eclampsia).

Which assessment findings by the nurse would require obtaining a blood glucose level on the newborn? 1. Jitteriness 2. Sucking on fingers 3. Lusty cry 4. Axillary temperature of 98°F

: 1 Explanation: 1. Jitteriness of the newborn is associated with hypoglycemia. Aggressive treatment is recommended after a single low blood glucose value if the infant shows this symptom.

A pregnant client is admitted to the emergency department with bleeding. The nurse realizes that the client might have placenta previa. Which signs would be suggestive of placenta previa? 1. Bright red vaginal bleeding 2. Sudden onset of vaginal bleeding 3. Firm and hard uterus 4. Change in the size of abdomen

1 Explanation: 1. As the lower uterine segment contracts and dilates, the placental villi are torn from the uterine wall, causing bright red bleeding.

Doppler flow studies (umbilical velocimetry) help to assess which of the following? 1. Placental function and sufficiency 2. Fetal heart rate 3. Fetal growth and fluid levels 4. Maturity of the fetal lungs

1 Explanation: 1. Doppler flow studies (umbilical velocimetry) help to assess placental function and sufficiency. Uteroplacental insufficiency is a risk for a woman with preeclampsia. If fetal growth restriction is present, Doppler velocimetry of the umbilical artery is useful for fetal surveillance.

) The client presents to the clinic for an initial prenatal examination. She asks the nurse whether there might be a problem for her baby because she has type B Rh-positive blood and her husband has type O Rh-negative blood, or because her sister's baby had ABO incompatibility. What is the nurse's best answer? Note: Credit will be given only if all correct choices and no incorrect choices are selected. Select all that apply. 1. "Your baby would be at risk for Rh problems if your husband were Rh-negative." 2. "Rh problems only occur when the mother is Rh-negative and the father is not." 3. "ABO incompatibility occurs only after the baby is born." 4. "We don't know for sure, but we can test for ABO incompatibility." 5. "Your husband's being type B puts you at risk for ABO incompatibility."

2, 3 Explanation: 2. Rh incompatibility is a possibility when the mother is Rh-negative and the father is Rh-positive. 3. ABO incompatibility is limited to type O mothers with a type A or B fetus and occurs after the baby is born.

The nurse assesses the gestational age of a newborn and informs the parents that the newborn is premature. Which of the following assessment findings is not congruent with prematurity? 1. Cry is weak and feeble 2. Clitoris and labia minora are prominent 3. Strong sucking reflex 4. Lanugo is plentiful

3 Explanation: 3. Poor suck, gag, and swallow reflexes are characteristic of a preterm newborn.

The nurse is caring for an infant born at 37 weeks that weighs 1750 g (3 pounds 10 ounces). The head circumference and length are in the 25th percentile. What statement would the nurse expect to find in the chart? 1. Preterm appropriate for gestational age, symmetrical IUGR 2. Term small for gestational age, symmetrical IUGR 3. Preterm small for gestational age, asymmetrical IUGR 4. Preterm appropriate for gestational age, asymmetrical IUGR

3 Explanation: 3. The infant is preterm at 37 weeks. Because the weight is below the 10th percentile, the infant is small for gestational age. Head circumference and length between the 10th and 90th percentiles indicate asymmetrical IUGR.

The nurse admits into the labor area a client who is in preterm labor. What assessment finding would constitute a diagnosis of preterm labor? 1. Cervical effacement of 30% or more 2. Cervical change of 0.5 cm per hour 3. 2 contractions in 30 minutes 4. 8 contractions in 1 hour

4 Explanation: 4. 8 contractions in a 60 minute period does define a diagnosis of preterm labor.

A 7 pound 14 ounce girl was born to an insulin-dependent type II diabetic mother 2 hours ago. The infant's blood sugar is 47 mg/dL. What is the best nursing action? 1. To recheck the blood sugar in 6 hours 2. To begin an IV of 10% dextrose 3. To feed the baby 1 ounce of formula 4. To document the findings in the chart

4 Explanation: 4. A blood sugar level of 47 mg/dL is a normal finding; documentation is an appropriate action.

) A woman's history and appearance suggest drug abuse. What is the nurse's best approach? 1. Ask the woman directly, "Do you use any street drugs?" 2. Ask the woman whether she would like to talk to a counselor. 3. Ask some questions about over-the-counter medications and avoid mention of illicit drugs. 4. Explain how harmful drugs can be for her baby.

: 1 Explanation: 1. If drug abuse is suspected, the nurse should ask direct questions and be matter-of-fact and nonjudgmental to elicit honest responses.

) Infants of women with preeclampsia during pregnancy tend to be small for gestational age (SGA) because of which condition? 1. Intrauterine growth restriction 2. Oliguria 3. Proteinuria 4. Hypertension

: 1 Explanation: 1. Infants of women with preeclampsia during pregnancy tend to be small for gestational age (SGA) because of intrauterine growth restriction. The cause is related specifically to maternal vasospasm and hypovolemia, which result in fetal hypoxia and malnutrition.

A woman is being treated for preterm labor with magnesium sulfate. The nurse is concerned that the client is experiencing early drug toxicity. What assessment finding by the nurse indicates early magnesium sulfate toxicity? 1. Patellar reflexes weak or absent 2. Increased appetite 3. Respiratory rate of 16 4. Fetal heart rate of 120

1 Explanation: 1. Early signs of magnesium sulfate toxicity are related to a decrease in deep tendon reflexes.

) The nurse is caring for the newborn of a diabetic mother whose blood glucose level is 39 mg/dL. What should the nurse include in the plan of care for this newborn? 1. Offer early feedings with formula or breast milk. 2. Provide glucose water exclusively. 3. Evaluate blood glucose levels at 12 hours after birth. 4. Assess for hypothermia.

1 Explanation: 1. IDMs whose serum glucose falls below 40 mg/dL should have early feedings with formula or breast milk (colostrum).

A newly diagnosed insulin-dependent type 1 diabetic with good blood sugar control is at 20 weeks' gestation. She asks the nurse how her diabetes will affect her baby. What would the best explanation include? 1. "Your baby could be smaller than average at birth." 2. "Your baby will probably be larger than average at birth." 3. "As long as you control your blood sugar, your baby will not be affected at all." 4. "Your baby might have high blood sugar for several days."

2 Explanation: 2. Characteristically, infants of mothers with diabetes are large for gestational age (LGA) as a result of high levels of fetal insulin production stimulated by the high levels of glucose crossing the placenta from the mother. Sustained fetal hyperinsulinism and hyperglycemia ultimately lead to excessive growth, called macrosomia, and deposition of fat.

The client at 30 weeks' gestation is admitted with painless late vaginal bleeding. The nurse understands that expectant management includes which of the following? 1. Limiting vaginal exams to only one per 24-hour period. 2. Evaluating the fetal heart rate with an internal monitor. 3. Monitoring for blood loss, pain, and uterine contractibility. 4. Assessing blood pressure every 2 hours.

3 Explanation: 3. Blood loss, pain, and uterine contractibility need to be assessed for client comfort and safety.

) The nurse is planning care for a preterm newborn. Which nursing diagnosis has the highest priority? 1. Tissue Integrity, Impaired 2. Infection, Risk for 3. Gas Exchange, Impaired 4. Family Processes, Dysfunctional

3 Explanation: 3. Gas Exchange, Impaired is related to immature pulmonary vasculature and inadequate surfactant production and has the highest priority.

A client in her second trimester is complaining of spotting. Causes for spotting in the second trimester are diagnosed primarily through the use of which of the following? 1. A nonstress test 2. A vibroacoustic stimulation test 3. An ultrasound 4. A contraction stress test

3 Explanation: 3. Indirect diagnosis is made by localizing the placenta via tests that require no vaginal examination. The most commonly employed diagnostic test is the transabdominal ultrasound scan.

) The client with insulin-dependent type 2 diabetes and an HbA1c of 5.0% is planning to become pregnant soon. What anticipatory guidance should the nurse provide this client? 1. Insulin needs decrease in the first trimester and usually begin to rise late in the first trimester as glucose use and glycogen storage by the woman and fetus increase. 2. The risk of ketoacidosis decreases during the length of the pregnancy. 3. Vascular disease that accompanies diabetes slows progression. 4. The baby is likely to have a congenital abnormality because of the diabetes.

: 1 Explanation: 1. Insulin needs decrease in the first trimester and usually begin to rise late in the first trimester as glucose use and glycogen storage by the woman and fetus increase.

) A client is admitted to the birth setting in early labor. She is 3 cm dilated, -2 station, with intact membranes, and FHR of 150 bpm. Her membranes rupture spontaneously, and the FHR drops to 90 bpm with variable decelerations. What would the nurse's initial response be? 1. Perform a vaginal exam 2. Notify the physician 3. Place the client in a left lateral position 4. Administer oxygen at 2 L per nasal cannula

: 1 Explanation: 1. Prolapsed umbilical cord can occur when the membranes rupture. The fetus is more likely to experience variable decelerations because the amniotic fluid is insufficient to keep pressure off the umbilical cord. A vaginal exam is the best way to confirm.

A laboring mother has recurrent late decelerations. At birth, the infant has a heart rate of 100, is not breathing, and is limp and bluish in color. What nursing action is best? 1. Begin chest compressions. 2. Begin direct tracheal suctioning. 3. Begin bag-and-mask ventilation. 4. Obtain a blood pressure reading.

: 3 Explanation: 3. Most newborns can be effectively resuscitated by bag-and-mask ventilation.

) The nurse is caring for the newborn of a diabetic mother. Which of the following should be included in the nurse's plan of care for this newborn? 1. Offer early feedings. 2. Administer an intravenous infusion of glucose. 3. Assess for hypercalcemia. 4. Assess for hyperbilirubinemia immediately after birth.

1 Explanation: 1. Newborns of diabetic mothers may benefit from early feeding as they are extremely valuable in maintaining normal metabolism and lowering the possibility of such complications as hypoglycemia and hyperbilirubinemia.

The mother of a premature newborn questions why a gavage feeding catheter is placed in the mouth of the newborn and not in the nose. What is the nurse's best response? 1. "Most newborns are nose breathers." 2. "The tube will elicit the sucking reflex." 3. "A smaller catheter is preferred for feedings." 4. "Most newborns are mouth breathers."

1 Explanation: 1. Orogastric insertion is preferable to nasogastric because most infants are obligatory nose breathers.

The nurse has received end-of-shift reports in the high-risk maternity unit. Which client should the nurse see first? 1. The client at 26 weeks' gestation with placenta previa experiencing blood on toilet tissue after a bowel movement 2. The client at 30 weeks' gestation with placenta previa whose fetal monitor strip shows late decelerations 3. The client at 35 weeks' gestation with grade I abruptio placentae in labor who has a strong urge to push 4. The client at 37 weeks' gestation with pregnancy-induced hypertension whose membranes ruptured spontaneously

1 Explanation: 1. Assessment of the woman with placenta previa must be ongoing to prevent or treat complications that are potentially lethal to the mother and fetus. Painless, bright red vaginal bleeding is the best diagnostic sign of placenta previa. This client is the highest priority.

The nurse is evaluating the effectiveness of phototherapy on a newborn. Which evaluation indicates a therapeutic response to phototherapy? 1. The newborn maintains a normal temperature 2. An increase of serum bilirubin levels 3. Weight loss 4. Skin blanching yellow

1 Explanation: 1. Maintenance of temperature is an important aspect of phototherapy because the newborn is naked except for a diaper during phototherapy. The isolette helps the infant maintain his or her temperature while undressed.

The nurse is admitting a client for a cerclage procedure. The client asks for information about the procedure. What is the nurse's most accurate response? 1. "A stitch is placed in the cervix to prevent a spontaneous abortion or premature birth." 2. "The procedure is done during the third trimester." 3. "Cerclage is always placed after the cervix has dilated and effaced." 4. "An uncomplicated elective cerclage may is done on inpatient basis."

1 Explanation: 1. This is the correct description of cerclage.

When planning care for the premature newborn diagnosed with respiratory distress syndrome, which potential complications would the nurse anticipate? Note: Credit will be given only if all correct choices and no incorrect choices are selected. Select all that apply. 1. Hypoxia 2. Respiratory alkalosis 3. Metabolic acidosis 4. Massive atelectasis 5. Pulmonary edema

1, 3, 4, 5 Explanation: 1. The physiologic alterations of RDS can produce hypoxia as a complication. As a result of hypoxia, the pulmonary vasculature constricts, pulmonary vascular resistance increases, and pulmonary blood flow is reduced. 3. The physiologic alterations of RDS can produce metabolic acidosis as a complication. Because cells lack oxygen, the newborn begins an anaerobic pathway of metabolism, with an increase in lactate levels and a resulting base deficit. 4. The physiologic alterations of RDS can produce massive atelectasis as a complication. Upon expiration, the instability increases the atelectasis, which causes hypoxia and acidosis because of the lack of gas exchange. 5. The physiologic alterations of RDS can produce pulmonary edema as a complication. Opacification of the lungs on X-ray image may be due to massive atelectasis, diffuse alveolar infiltrate, or pulmonary edema.

) The nurse is preparing an educational in-service presentation about jaundice in the newborn. What content should the nurse include in this presentation? Note: Credit will be given only if all correct choices and no incorrect choices are selected. Select all that apply. 1. Physiologic jaundice occurs after 24 hours of age. 2. Pathologic jaundice occurs after 24 hours of age. 3. Phototherapy increases serum bilirubin levels. 4. The need for phototherapy depends on the bilirubin level and age of the infant. 5. Kernicterus causes irreversible neurological damage.

1, 5 Explanation: 1. Physiologic or neonatal jaundice is a normal process that occurs during transition from intrauterine to extrauterine life and appears after 24 hours of life. 5. Kernicterus refers to the deposition of unconjugated bilirubin in the basal ganglia of the brain and to permanent neurologic sequelae of untreated hyperbilirubinemia.

) The nurse is assessing a client who has severe preeclampsia. What assessment finding should be reported to the physician? 1. Excretion of less than 300 mg of protein in a 24-hour period 2. Platelet count of less than 100,000/mm3 3. Urine output of 50 mL per hour 4. 12 respirations

2 Explanation: 2. HELLP syndrome (hemolysis, elevated liver enzymes, and low platelet count) complicates 10% to 20% of severe preeclampsia cases and develops prior to 37 weeks' gestation 50% of the time. Vascular damage is associated with vasospasm, and platelets aggregate at sites of damage, resulting in low platelet count (less than 100,000/mm3).

Mild or chronic anemia in an infant may be treated adequately which of the following? 1. Transfusions with O-negative or typed and cross-matched packed red cells 2. Iron supplements or iron-fortified formulas 3. Steroid therapy 4. Antibiotics or antivirals

2 Explanation: 2. Mild or chronic anemia in an infant may be treated adequately with iron supplements or iron-fortified formulas.

The nurse notes that a 36-hour-old newborn's serum bilirubin level has increased from 14 mg/dL to 16.6 mg/dL in an 8-hour period. What nursing intervention would be included in the plan of care for this newborn? 1. Continue to observe 2. Begin phototherapy 3. Begin blood exchange transfusion 4. Stop breastfeeding

2 Explanation: 2. Neonatal hyperbilirubinemia must be considered pathologic if the serum bilirubin concentration is rising by more than 0.2 mg/dL per hour. If the newborn is over 24 hours old, which is past the time where an increase in bilirubin would result from pathologic causes, phototherapy may be the treatment of choice to prevent the possible complications of kernicterus.

A client was admitted to the labor area at 5 cm with ruptured membranes about 14 hours ago. What assessment data would be most beneficial for the nurse to collect? 1. Blood pressure 2. Temperature 3. Pulse 4. Respiration

2 Explanation: 2. Rupture of membranes places the mother at risk for infection. The temperature is the primary and often the first indication of a problem.

The nurse is assessing a 2-hour-old newborn delivered by cesarean at 38 weeks. The amniotic fluid was clear. The mother had preeclampsia. The newborn has a respiratory rate of 80, is grunting, and has nasal flaring. What is the most likely cause of this infant's condition? 1. Meconium aspiration syndrome 2. Transient tachypnea of the newborn 3. Respiratory distress syndrome 4. Prematurity of the neonate

2 Explanation: 2. The infant is term and was born by cesarean, and is most likely experiencing transient tachypnea of the newborn.

The client with blood type O Rh-negative has given birth to an infant with blood type O Rh-positive. The infant has become visibly jaundiced at 12 hours of age. The mother asks why this is happening. What is the best response by the nurse? 1. "The RhoGAM you received at 28 weeks' gestation did not prevent alloimmunization." 2. "Your body has made antibodies against the baby's blood that are destroying her red blood cells." 3. "The red blood cells of your baby are breaking down because you both have type O blood." 4. "Your baby's liver is too immature to eliminate the red blood cells that are no longer needed."

2 Explanation: 2. This explanation is accurate and easy for the client to understand. Newborns of Rh-negative and O blood type mothers are carefully assessed for blood type status, appearance of jaundice, and levels of serum bilirubin.

) The nurse is working with a pregnant adolescent. The client asks the nurse how the baby's condition is determined during labor. The nurse's best response is that during labor, the nurse will do which of the following? 1. Check the client's cervix by doing a pelvic exam every 2 hours. 2. Assess the fetus's heart rate with an electronic fetal monitor. 3. Look at the color and amount of bloody show that the client has. 4. Verify that the client's contractions are strong but not too close together.

2 Explanation: 2. This statement best answers the question the client has asked.

) The nurse is doing preconception counseling with a 28-year-old woman with no prior pregnancies. Which statement made by the client indicates to the nurse that the client has understood the teaching? 1. "I can continue to drink alcohol until I am diagnosed as pregnant." 2. "I need to stop drinking alcohol completely when I start trying to get pregnant." 3. "A beer once a week will not damage the fetus." 4. "I can drink alcohol while breastfeeding because it doesn't pass into breast milk."

2 Explanation: 2. Women should discontinue drinking alcohol when they start to attempt to become pregnant due to possible effects of alcohol on the fetus.

The nurse is assessing the newborn for symptoms of anemia. If the blood loss is acute, the baby may exhibit which of the following signs of shock? Note: Credit will be given only if all correct choices and no incorrect choices are selected. Select all that apply. 1. Increased pulse 2. High blood pressure 3. Tachycardia 4. Bradycardia 5. Capillary filling time greater than 3 seconds

3, 5 Explanation: 3. Tachycardia would be a sign of shock. 5. Capillary filling time greater than 3 seconds would be a sign of shock.

) The parents of a newborn have just been told their infant has tetralogy of Fallot. The parents do not seem to understand the explanation given by the physician. What statement by the nurse is best? 1. "With this defect, not enough of the blood circulates through the lungs, leading to a lack of oxygen in the baby's body." 2. "The baby's aorta has a narrowing in a section near the heart that makes the left side of the heart work harder." 3. "The blood vessels that attach to the ventricles of the heart are positioned on the wrong sides of the heart." 4. "Your baby's heart doesn't circulate blood well because the left ventricle is smaller and thinner than normal."

: 1 Explanation: 1. Tetralogy of Fallot is a cyanotic heart defect that comprises four abnormalities: pulmonary stenosis, ventricular septal defect, overriding aorta, and right ventricle hypertrophy. The severity of symptoms depends on the degree of pulmonary stenosis, the size of the ventricular septal defect, and the degree to which the aorta overrides the septal defect.

) A nursing instructor is demonstrating how to perform a heel stick on a newborn. To obtain an accurate capillary hematocrit reading, what does the nursing instructor tell the student do? 1. Rub the heel vigorously with an isopropyl alcohol swab prior to obtaining blood. 2. Use a previous puncture site. 3. Cool the heel prior to obtaining blood. 4. Use a sterile needle and aspirate

: 1 Explanation: 1. The site should be cleaned by rubbing vigorously with 70% isopropyl alcohol swab. The friction produces local heat, which aids vasodilation.

The pregnant client at 41 weeks is scheduled for labor induction. She asks the nurse whether induction is really necessary. What response by the nurse is best? 1. "Babies can develop postmaturity syndrome, which increases their chances of having complications after birth." 2. "When infants are born 2 or more weeks after their due date, they have meconium in the amniotic fluid." 3. "Sometimes the placenta ages excessively, and we want to take care of that problem before it happens." 4. "The doctor wants to be proactive in preventing any problems with your baby if he gets any bigger."

: 1 Explanation: 1. The term postmaturity applies to the infant who is born after 42 completed weeks of gestation and demonstrates characteristics of postmaturity syndrome.

The nurse is observing a student nurse care for a neonate undergoing intensive phototherapy. Which action by the student nurse indicates an understanding of how to provide this care? 1. Urine specific gravity is assessed each voiding. 2. Eye coverings are left off to help keep the baby calm. 3. Temperature is checked every 6 hours. 4. The infant is taken out of the isolette for diaper changes.

: 1 Explanation: 1. This action is correct. Specific gravity provides one measure of urine concentration. Highly concentrated urine is associated with a dehydrated state. Weight loss is also a sign of developing dehydration in the newborn.

) The labor nurse is caring for a client at 38 weeks' gestation who has been diagnosed with symptomatic placenta previa. Which physician order should the nurse question? 1. Begin oxytocin drip rate at 0.5 milliunits/min. 2. Assess fetal heart rate every 10 minutes. 3. Weigh all vaginal pads. 4. Assess hematocrit and hemoglobin.

: 1 Explanation: 1. This order should be questioned, as this client is not a good candidate for labor induction.

The nurse is caring for a 2-hour-old newborn whose mother is diabetic. The nurse assesses that the newborn is experiencing tremors. Which nursing action has the highest priority? 1. Obtain a blood calcium level. 2. Take the newborn's temperature. 3. Obtain a bilirubin level. 4. Place a pulse oximeter on the newborn.

: 1 Explanation: 1. Tremors are a sign of hypocalcemia. Diabetic mothers tend to have decreased serum magnesium levels at term. This could cause secondary hypoparathyroidism in the infant.

During newborn resuscitation, how does the nurse evaluate the effectiveness of bag-and-mask ventilations? 1. The rise and fall of the chest 2. Sudden wakefulness 3. Urinary output 4. Adequate thermoregulation

: 1 Explanation: 1. With proper resuscitation, chest movement is observed for proper ventilation. Pressure should be adequate to move the chest wall.

In caring for the premature newborn, the nurse must assess hydration status continually. Assessment parameters should include which of the following? Note: Credit will be given only if all correct choices and no incorrect choices are selected. Select all that apply. 1. Volume of urine output 2. Weight 3. Blood pH 4. Head circumference 5. Bowel sounds

: 1, 2 Explanation: 1. In order to assess hydration status, volume of urine output must be evaluated. 2. In order to assess hydration status, the infant's weight must be evaluated.

A client is admitted to the labor and delivery unit in active labor. What nursing diagnoses might apply to the client with suspected abruptio placentae? Note: Credit will be given only if all correct choices and no incorrect choices are selected. Select all that apply. 1. Fluid Volume, Deficient, Risk for, related to hypovolemia secondary to excessive blood loss 2. Tissue Perfusion: Peripheral, Ineffective, related to blood loss secondary to uterine atony following birth 3. Anxiety related to concern for own personal status and the baby's safety 4. Knowledge, Deficient related to lack of information about inherited genetic defects 5. Alteration in Respiratory Function related to blood loss

: 1, 2, 3 Explanation: 1. Maternal and perinatal fetal mortality are concerns due to hypoxia. 2. Maternal and perinatal fetal mortality are concerns due to blood loss. 3. This mother would be anxious for herself and her baby.

) The nurse is presenting a class on preterm labor, its causes, and treatments to a group of newly pregnant couples. Which statements regarding preterm labor are true? Note: Credit will be given only of all correct choices and no incorrect choices are selected. Select all that apply. 1. Antepartum hemorrhage can cause preterm labor. 2. Trauma can cause preterm labor. 3. Infection can cause preterm labor. 4. Magnesium sulfate is a drug used to stop contractions. 5. Sedatives and narcotics may be given to stop labor.

: 1, 2, 3, 4 Explanation: 1. Hemorrhage from placenta previa or abruption can cause preterm labor. 2. Trauma to the abdomen or uterus can cause preterm labor. 3. Infections such as urinary tract infections can cause preterm labor. 4. Magnesium sulfate acts as a CNS depressant by decreasing the quantity of acetylcholine released by motor nerve impulses and thereby blocking neuromuscular transmission.

The nurse knows that a mother who has been treated for Beta streptococcus passes this risk on to her newborn. Risk factors for neonatal sepsis caused by Beta streptococcus include which of the following? Note: Credit will be given only if all correct choices and no incorrect choices are selected. Select all that apply. 1. Prematurity 2. Maternal intrapartum fever 3. Membranes ruptured for longer than 18 hours 4. A previously infected infant with GBS disease 5. An older mother having her first baby

: 1, 2, 3, 4 Explanation: 1. Prematurity is a risk factor. 2. Maternal intrapartum fever is a risk factor. 3. Prolonged rupture of membranes is a risk factor. 4. A previously infected infant increases the risk.

The nurse caring for a newborn with anemia would expect which initial laboratory data to be included in the initial assessment? Note: Credit will be given only if all correct choices and no incorrect choices are selected. Select all that apply. 1. Hemoglobin 2. Hematocrit 3. Reticulocyte count 4. Direct Coombs' test 5. Cord serum OgM

: 1, 2, 3, 4 Explanation: 1. The initial laboratory workup for anemia should include hemoglobin measurements. 2. The initial laboratory workup for anemia should include hematocrit measurements. 3. The initial laboratory workup for anemia should include a reticulocyte count. 4. The direct Coombs' test reveals the presence of antibody-coated (sensitized) Rh-positive red blood cells in the newborn and should be included in the initial laboratory workup for anemia.

The nurse is caring for an infant who was delivered in a car on the way to the hospital and who has developed cold stress. Which finding requires immediate intervention? 1. Increased skin temperature and respirations 2. Blood glucose level of 45 3. Room-temperature IV running 4. Positioned under radiant warmer

: 3 Explanation: 3. IV fluids should be warmed prior to administration and the newborn can be wrapped in a chemically activated warming mattress immediately following birth to decrease the postnatal fall in temperature that normally occurs.

The nurse is working with a woman who abuses stimulants. The nurse is aware that the fetus is at risk for which of the following? Note: Credit will be given only if all correct and no incorrect choices are selected. Select all that apply. 1. Withdrawal symptoms 2. Cardiac anomalies 3. Sudden infant death syndrome 4. Being small for gestational age 5. Fetal alcohol syndrome

: 1, 2, 3, 4 Explanation:1. Infants born to mothers who abuse stimulants such as amphetamines can have withdrawal symptoms. 2. Infants born to mothers who abuse stimulants such as cocaine can be born with cardiac anomalies. 3. Infants born to mothers who abuse stimulants such as cocaine can have sudden infant death syndrome. 4. Infants born to mothers who abuse stimulants such as nicotine can be small for gestational age.

When blood pressure and other signs indicate that the preeclampsia is worsening, hospitalization is necessary to monitor the woman's condition closely. At that time, which of the following should be assessed? Note: Credit will be given only if all correct choices and no incorrect choices are selected. Select all that apply. 1. Fetal heart rate 2. Blood pressure 3. Temperature 4. Urine color 5. Pulse and respirations

: 1, 2, 3, 5 Explanation: 1. Determine the fetal heart rate along with blood pressure, or monitor continuously with the electronic fetal monitor if the situation indicates. 2. Determine blood pressure every 1 to 4 hours, or more frequently if indicated by medication or other changes in the woman's status. 3. Determine temperature every 4 hours, or every 2 hours if elevated or if premature rupture of the membranes (PROM) has occurred. 5. Determine pulse rate and respirations along with blood pressure.

The nurse is presenting a class for nursing students on multiple-gestation pregnancy. Which statements about multiple-gestation pregnancies are accurate? Note: Credit will be given only if all correct choices and no incorrect choices are selected. Select all that apply. 1. Hypertension is a major maternal complication. 2. Gestational diabetes occurs more often. 3. Maternal anemia occurs. 4. Pulmonary embolism is 12 times more likely to develop during pregnancy with multiple gestations. 5. Multiple gestations are more likely to acquire HELLP.

: 1, 2, 3, 5 Explanation: 1. Hypertension is a complication in multiple-gestation pregnancies. 2. Gestational diabetes occurs more often in multiple gestations. 3. Maternal anemia occurs because of demands of the multiple gestations. 5. Multiple gestations are more likely to acquire HELLP (hemolytic anemia, elevated liver enzymes, and low platelet count) syndrome, a complication resulting from eclampsia or preeclampsia.

The nurse is caring for a newborn with jaundice. The parents question why the newborn is not under phototherapy lights. The nurse explains that the fiber-optic blanket is beneficial because of which of the following? Note: Credit will be given only if all correct choices and no incorrect choices are selected. Select all that apply. 1. Lights can stay on all the time. 2. The eyes do not need to be covered. 3. The lights will need to be removed for feedings. 4. Newborns do not get overheated. 5. Weight loss is not a complication of this system.

: 1, 2, 4, 5 Explanation: 1. With the fiber-optic blanket, the light stays on at all times. 2. The eyes do not have to be covered with a fiber optic blanket. 4. With the fiber-optic blanket, greater surface area is exposed and there are no thermoregulation issues. 5. Fluid and weight loss are not complications of fiber-optic blankets.

Which assessment findings would lead the nurse to suspect that a newborn might have a congenital heart defect? Note: Credit will be given only if all correct choices and no incorrect choices are selected. Select all that apply. 1. Cyanosis 2. Heart murmur 3. Bradycardia 4. Low urinary outputs 5. Tachypnea

: 1, 3, 4, 5 Explanation: 1. Central cyanosis is defined as a visible, blue discoloration of the skin caused by decreased oxygen saturation levels and is a common manifestation of a cardiac defect. 3. The signs of congestive heart failure include tachycardia, not bradycardia. 4. The signs of congestive heart failure include low urinary output. 5. The signs of congestive heart failure include tachypnea.

The nurse has admitted a woman with cervical insufficiency. The nurse is aware that causes of this condition include which of the following? Note: Credit will be given only if all correct choices and no incorrect choices are selected. Select all that apply. 1. Congenital factors 2. Intercourse during pregnancy 3. Infection 4. Increased uterine volume 5. Past cervical surgeries

: 1, 3, 4, 5 Explanation: 1. Congenitally incompetent cervix may be found in women exposed to diethylstilbestrol (DES) or those with a bicornuate uterus. 3. Infection or trauma can cause acquired cervical incompetence. 4. Cervical insufficiency can occur in multiple-gestation pregnancies. 5. Previous elective abortion or cervical manipulation can lead to cervical insufficiency.

) The nurse is caring for a prenatal client. Reviewing the client's pregnancy history, the nurse identifies risk factors for an at-risk newborn, including which of the following? Note: Credit will be given only if all correct choices and no incorrect choices are selected. Select all that apply. 1. The mother's low socioeconomic status 2. Maternal age of 26 3. Mother's exposure to toxic chemicals 4. More than three previous deliveries 5. Maternal hypertension

: 1, 3, 4, 5 Explanation: 1. Low socioeconomic status is associated with at-risk newborns. 3. Exposure to environmental dangers, such as toxic chemicals is associated with at-risk newborns. 4. Maternal factors such as multiparity are associated with at-risk newborns. 5. Preexisting maternal conditions, such as heart disease, diabetes, hypertension, hyperthyroidism, and renal disease are associated with at-risk newborns.

Which fetal/neonatal risk factors would lead the nurse to anticipate a potential need to resuscitate a newborn? Note: Credit will be given only if all correct choices and no incorrect choices are selected. Select all that apply. 1. Nonreassuring fetal heart rate pattern/sustained bradycardia 2. Fetal scalp/capillary blood sample pH greater than 7.25 3. History of meconium in amniotic fluid 4. Prematurity 5. Significant intrapartum bleeding

: 1, 3, 4, 5 Explanation: 1. Nonreassuring fetal heart rate pattern/sustained bradycardia would be considered a potential need to resuscitate a newborn. 3. History of meconium in amniotic fluid would be considered a potential need to resuscitate a newborn. 4. Prematurity would be considered a potential need to resuscitate a newborn. 5. Significant intrapartum bleeding would be considered a potential need to resuscitate a newborn.

) If the woman is Rh negative and not sensitized, she is given Rh immune globulin to prevent what? 1. The potential for hemorrhage 2. Hyperhomocysteinemia 3. Antibody formation 4. Tubal pregnancy

: 3 Explanation: 3. If the woman is Rh negative and not sensitized, she is given Rh immune globulin to prevent antibody formation.

) The nurse is caring for an infant of a diabetic mother. Which potential complications would the nurse consider in planning care for this newborn? Note: Credit will be given if all correct choices and no incorrect choices are selected. Select all that apply. 1. Tremors 2. Hyperglycemia 3. Hyperbilirubinemia 4. Respiratory distress syndrome 5. Birth trauma

: 1, 3, 4, 5 Explanation: 1. Tremors are a clinical sign of hypocalcemia. 3. Hyperbilirubinemia is caused by slightly decreased extracellular fluid volume, which increases the hematocrit level. 4. Respiratory distress syndrome (RDS) is a complication that occurs more frequently in newborns of diabetic mothers whose diabetes is not well controlled. 5. Because most IDMs are macrosomic, trauma may occur during labor and vaginal birth resulting in shoulder dystocia, brachial plexus injuries, subdural hemorrhage, cephalohematoma, and asphyxia.

A client at 32 weeks' gestation is admitted with painless vaginal bleeding. Placenta previa has been confirmed by ultrasound. What should be included in the nursing plan? Note: Credit will be given only if all correct choices and no incorrect choices are selected. Select all that apply. 1. No vaginal exams 2. Encouraging activity 3. No intravenous access until labor begins 4. Evaluating fetal heart rate with an external monitor 5. Monitoring blood loss, pain, and uterine contractility

: 1, 4, 5 Explanation: 1. Expectant management of placenta previa is made by localizing the placenta via tests that require no vaginal examination. 4. Expectant management of placenta previa, when the client is at less than 37 weeks' gestation, includes evaluating FHR with an external monitor. 5. Expectant management of placenta previa, when the client is at less than 37 weeks' gestation, includes monitoring blood loss, pain, and uterine contractility.

A client is admitted to the labor suite. It is essential that the nurse assess the woman's status in relation to which infectious diseases? Note: Credit will be given only if all correct choices and no incorrect choices are selected. Select all that apply. 1. Chlamydia trachomatis 2. Rubeola 3. Varicella 4. Group B streptococcus 5. Acute pyelonephritis

: 1, 4, 5 Explanation: 1. The infant may develop chlamydial pneumonia and Chlamydia trachomatis may be responsible for premature labor and fetal death. Chlamydial infection should be assessed. 4. Women may transmit GBS to their fetus in utero or during childbirth. GBS is a leading infectious cause of neonatal sepsis and mortality and should be assessed. 5. Acute pyelonephritis should be assessed as there is an increased risk of premature birth and intrauterine growth restriction (IUGR).

A client in labor is found to have meconium-stained amniotic fluid upon rupture of membranes. At delivery, the nurse finds the infant to have depressed respirations and a heart rate of 80. What does the nurse anticipate? 1. Delivery of the neonate on its side with head up, to facilitate drainage of secretions. 2. Direct tracheal suctioning by specially trained personnel. 3. Preparation for the immediate use of positive pressure to expand the lungs. 4. Suctioning of the oropharynx when the newborn's head is delivered.

: 2 Explanation: 2. If the infant has absent or depressed respirations, heart rate less than 100 beats/min, or poor muscle tone, direct tracheal suctioning by specially trained personnel is recommended.

The nurse is caring for a jaundiced infant receiving bank light phototherapy in an isolette. Which finding requires an immediate intervention? 1. Eyes are covered, no clothing on, diaper in place 2. Axillary temperature 99.7°F 3. Infant removed from the isolette for breastfeeding 4. Loose bowel movement

: 2 Explanation: 2. Temperature assessment is indicated to detect hypothermia or hyperthermia. Normal temperature ranges are 97.7°F-98.6°F. Vital signs should be monitored every 4 hours with axillary temperatures.

) A 26-year-old client is 28 weeks pregnant. She has developed gestational diabetes. She is following a program of regular exercise, which includes walking, bicycling, and swimming. What instructions should be included in a teaching plan for this client? 1. "Exercise either just before meals or wait until 2 hours after a meal." 2. "Carry hard candy (or other simple sugar) when exercising." 3. "If your blood sugar is 120 mg/dL, eat 20 g of carbohydrate." 4. "If your blood sugar is more than 120 mg/dL, drink a glass of whole milk."

: 2 Explanation: 2. The nurse should advise her to carry a simple sugar such as hard candy because of the possibility of exercise-induced hypoglycemia.

) During the nursing assessment of a woman with ruptured membranes, the nurse suspects a prolapsed umbilical cord. What would the nurse's priority action be? 1. To help the fetal head descend faster 2. To use gravity and manipulation to relieve compression on the cord 3. To facilitate dilation of the cervix with prostaglandin gel 4. To prevent head compression

: 2 Explanation: 2. The top priority is to relieve compression on the umbilical cord to allow blood flow to reach the fetus. It is because some obstetric maneuvers to relieve cord compression are complicated that cesarean birth is sometimes necessary.

The prenatal clinic nurse is caring for a client with hyperemesis gravidarum at 14 weeks' gestation. The vital signs are: blood pressure 95/48, pulse 114, respirations 24. Which order should the nurse implement first? 1. Weigh the client. 2. Give 1 liter of lactated Ringer's solution IV. 3. Administer 30 mL Maalox (magnesium hydroxide) orally. 4. Encourage clear liquids orally.

: 2 Explanation: 2. The vital signs indicate hypovolemia from dehydration, which leads to hypotension and increased pulse rate. Giving this client a liter of lactated Ringer's solution intravenously will reestablish vascular volume and bring the blood pressure up, and the pulse and respiratory rate down.

The nurse is planning an in-service educational program to talk about disseminated intravascular coagulation (DIC). The nurse should identify which conditions as risk factors for developing DIC? Note: Credit will be given only if all correct choices and no incorrect choices are selected. Select all that apply. 1. Diabetes mellitus 2. Abruptio placentae 3. Fetal demise 4. Multiparity 5. Preterm labor

: 2, 3 Explanation: 2. As a result of the damage to the uterine wall and the retroplacental clotting with covert abruption, large amounts of thromboplastin are released into the maternal blood supply, which in turn triggers the development of disseminated intravascular coagulation (DIC) and the resultant hypofibrinogenemia. 3. Perinatal mortality associated with abruptio placentae is approximately 25%. If fetal hypoxia progresses unchecked, irreversible brain damage or fetal demise may result.

The nurse is assessing a newborn diagnosed with physiologic jaundice. Which findings would the nurse expect? Note: Credit will be given only if all correct choices and no incorrect choices are selected. Select all that apply. 1. Jaundice present within the first 24 hours of life 2. Appearance of jaundice symptoms after 24 hours of life 3. Yellowish coloration of the sclera of the eyes 4. Cephalohematoma or excessive bruising 5. Cyanosis

: 2, 3 Explanation: 2. Physiologic or neonatal jaundice is a normal process that occurs during transition from intrauterine to extrauterine life and appears after 24 hours of life. 3. Jaundice is a yellowish coloration of the skin and sclera of the eyes that develops from the deposit of yellow pigment bilirubin in lipid/fat-containing tissues.

Which nursing interventions are appropriate when caring for the newborn undergoing phototherapy? Note: Credit will be given only if all correct choices and no incorrect choices are selected. Select all that apply. 1. Cover the newborn's eyes at all times, even when not under the lights. 2. Close the newborn's eyelids before applying eye patches. 3. Inspect the eyes each shift for conjunctivitis. 4. Keep the baby swaddled in a blanket to prevent heat loss. 5. Reposition the baby every 2 hours.

: 2, 3, 5 Explanation: 2. Apply eye patches over the newborn's closed eyes during exposure to banks of phototherapy. 3. Discontinue conventional phototherapy and remove the eye patches at least once per shift to assess the eyes for the presence of conjunctivitis. 5. Repositioning allows equal exposure of all skin areas and prevents pressure areas.

Which of the following are considered risk factors for development of severe hyperbilirubinemia? Note: Credit will be given only if all correct choices and no incorrect choices are selected. Select all that apply. 1. Northern European descent 2. Previous sibling received phototherapy 3. Gestational age 27 to 30 weeks 4. Exclusive breastfeeding 5. Infection

: 2, 4, 5 Explanation: 2. Previous sibling received phototherapy is considered a risk factor for development of severe hyperbilirubinemia. 4. Exclusive breastfeeding, particularly if nursing is not going well and excessive weight loss is experienced, is considered a risk factor for development of severe hyperbilirubinemia. 5. Infection is considered a risk factor for development of severe hyperbilirubinemia.

A pregnant client has been admitted with a diagnosis of hyperemesis. Which orders written by the primary healthcare provider are the highest priorities for the nurse to implement? Note: Credit will be given only if all correct choices and no incorrect choices are selected. Select all that apply. 1. Obtain complete blood count. 2. Start intravenous fluid with multivitamins. 3. Check admission weight. 4. Obtain urine for urinalysis. 5. Give a medication to stop the nausea and vomiting.

: 2, 5 Explanation: 2. Starting intravenous fluid with multivitamins is a priority if the client has been vomiting. 5. Giving a medication to stop the nausea and vomiting is a priority.

) Whether sensitization is the result of a blood transfusion or maternal-fetal hemorrhage for any reason, what test can be performed to determine the amount of Rh(D) positive blood present in the maternal circulation and to calculate the amount of Rh immune globulin needed? 1. Indirect Coombs' test 2. Nonstress test 3. Kleihauer-Betke or rosette test 4. Direct Coombs' test

: 3 Explanation: 3. A Kleihauer-Betke or rosette test can be performed to determine the amount of Rh(D) positive blood present in the maternal circulation and to calculate the amount of Rh immune globulin needed.

A nurse explains to new parents that their newborn has developed respiratory distress syndrome (RDS). Which of the following signs and symptoms would not be characteristic of RDS? 1. Grunting respirations 2. Nasal flaring 3. Respiratory rate of 40 during sleep 4. Chest retractions

: 3 Explanation: 3. A respiratory rate of 40 during sleep is normal.

) A woman is experiencing preterm labor. The client asks why she is on betamethasone. Which is the nurse's best response? 1. "This medication will halt the labor process until the baby is more mature." 2. "This medication will relax the smooth muscles in the infant's lungs so the baby can breathe." 3. "This medication is effective in stimulating lung development in the preterm infant." 4. "This medication is an antibiotic that will treat your urinary tract infection, which caused preterm labor."

: 3 Explanation: 3. Betamethasone or dexamethasone is often administered to the woman whose fetus has an immature lung profile to promote fetal lung maturation.

The nurse is supervising care in the emergency department. Which situation most requires an intervention? 1. Moderate vaginal bleeding at 36 weeks' gestation; client has an IV of lactated Ringer's solution running at 125 mL/hour 2. Spotting of pinkish-brown discharge at 6 weeks' gestation and abdominal cramping; ultrasound scheduled in 1 hour 3. Bright red bleeding with clots at 32 weeks' gestation; pulse = 110, blood pressure 90/50, respirations = 20 4. Dark red bleeding at 30 weeks' gestation with normal vital signs; client reports an absence of fetal movement

: 3 Explanation: 3. Bleeding in the third trimester is usually a placenta previa or placental abruption. Observe the woman for indications of shock, such as pallor, clammy skin, perspiration, dyspnea, or restlessness. Monitor vital signs, particularly blood pressure and pulse, for evidence of developing shock.

Which maternal-child client should the nurse see first? 1. Blood type O, Rh-negative 2. Indirect Coombs' test negative 3. Direct Coombs' test positive 4. Blood type B, Rh-positive

: 3 Explanation: 3. Direct Coombs' test is done on the infant's blood to detect antibody-coated Rh-positive RBCs. If the mother's indirect Coombs' test is positive and her Rh-positive infant has a positive direct Coombs' test, Rh immune globulin is not given; in this case, the infant is carefully monitored for hemolytic disease.

) A client who is 11 weeks pregnant presents to the emergency department with complaints of dizziness, lower abdominal pain, and right shoulder pain. Laboratory tests reveal a beta-hCG at a lower-than-expected level for this gestational age. An adnexal mass is palpable. Ultrasound confirms no intrauterine gestation. The client is crying and asks what is happening. The nurse knows that the most likely diagnosis is an ectopic pregnancy. Which statement should the nurse include? 1. "You're feeling dizzy because the pregnancy is compressing your vena cava." 2. "The pain is due to the baby putting pressure on nerves internally." 3. "The baby is in the fallopian tube; the tube has ruptured and is causing bleeding." 4. "This is a minor problem. The doctor will be right back to explain it to you."

: 3 Explanation: 3. The woman who experiences one-sided lower abdominal pain or diffused lower abdominal pain, vasomotor disturbances such as fainting or dizziness, and referred right shoulder pain from blood irritating the subdiaphragmatic phrenic nerve is experiencing an ectopic pregnancy.

The nurse educator is describing the different kinds of abruptio placentae to a group of students, explaining that in a complete abruptio placentae, which of the following occurs? 1. Separation begins at the periphery of the placenta. 2. The placenta separates centrally and blood is trapped between the placenta and the uterine wall. 3. There is massive vaginal bleeding in the presence of almost total separation. 4. Blood passes between the fetal membranes and the uterine wall, and escapes vaginally.

: 3 Explanation: 3. There is massive vaginal bleeding in the presence of almost total separation describes a complete separation of the placenta.

) A diabetic client goes into labor at 36 weeks' gestation. Provided that tests for fetal lung maturity are successful, the nurse will anticipate which of the following interventions? Note: Credit will be given only if all correct and no incorrect choices are selected. Select all that apply. 1. Administration of tocolytic therapy 2. Beta-sympathomimetic administration 3. Allowance of labor to progress 4. Hourly blood glucose monitoring 5. Cesarean birth may be indicated if evidence of reassuring fetal status exists

: 3, 4 Explanation: 3. There will be no attempt to stop the labor, as this can compromise the mother and fetus. 4. To reduce incidence of congenital anomalies and other problems in the newborn, the woman should be euglycemic (have normal blood glucose) throughout the pregnancy.

) The nurse is analyzing assessment findings on four newborns. Which finding might suggest a congenital heart defect? 1. Apical heart rate of 140 beats per minute 2. Respiratory rate of 40 3. Temperature of 36.5°C 4. Visible, blue discoloration of the skin

: 4 Explanation: 4. Central cyanosis is defined as a visible, blue discoloration of the skin caused by decreased oxygen saturation levels and is a common manifestation of a cardiac defect.

A 26-year-old client is 26 weeks pregnant. Her previous births include two large-for-gestational-age babies and one unexplained stillbirth. Which tests would the nurse anticipate as being most definitive in diagnosing gestational diabetes? 1. A 50g, 1-hour glucose screening test 2. A single fasting glucose level 3. A 100g, 1-hour glucose tolerance test 4. A 100g, 3-hour glucose tolerance test

: 4 Explanation: 4. Gestational diabetes is diagnosed if two or more of the following values are met or exceeded after taking the 100 g, 3-hour OGTT: Fasting: 95 mg/dL; 1 hour: 180 mg/dL; 2 hours: 155 mg/dL; 3 hours: 140 mg/dL.

A newborn is receiving phototherapy. Which intervention by the nurse would be most important? 1. Measurement of head circumference 2. Encouraging the mother to stop breastfeeding 3. Stool blood testing 4. Assessment of hydration status

: 4 Explanation: 4. Infants undergoing phototherapy treatment have increased water loss and loose stools as a result of bilirubin excretion. This increases their risk of dehydration.

The client is carrying monochorionic-monoamniotic twins. The nurse teaches the client what this is, and the implications of this finding. The nurse knows that teaching is successful when the client states which of the following? 1. "My babies came from two eggs." 2. "About two thirds of twins have this amniotic sac formation." 3. "My use of a fertility drug led to this issue." 4. "My babies have a lower chance of surviving to term than fraternal twins do."

: 4 Explanation: 4. Monochorionic-monoamniotic twins are both in one amniotic sac. There is an increased risk of umbilical cords becoming tangled or knotted and a higher incidence of fetal demise.

) Intervention to reduce preterm birth can be divided into primary prevention and secondary prevention. What does secondary prevention include? 1. Diagnosis and treatment of infections 2. Cervical cerclage 3. Progesterone administration 4. Antibiotic treatment and tocolysis

: 4 Explanation: 4. Secondary prevention strategies are antibiotic treatment and tocolysis.

The nurse is caring for a client who was just admitted to rule out ectopic pregnancy. Which orders are the most important for the nurse to perform? Note: Credit will be given only if all correct choices and no incorrect choices are selected. Select all that apply. 1. Assess the client's temperature. 2. Document the time of the client's last meal. 3. Obtain urine for urinalysis and culture. 4. Report complaints of dizziness or weakness. 5. Have the lab draw blood for B-hCG level every 48 hours.

: 4, 5 Explanation: 4. Reporting complaints of dizziness and weakness is important, as it can indicate hypovolemia from internal bleeding. 5. Having the lab draw blood for B-hCG levels every 48 hours is important, as the level rises much more slowly in ectopic pregnancy than in normal pregnancy.

The nurse is caring for a 3-week-old preterm newborn born at 29 weeks of gestation. While taking vital signs and changing the newborn's diaper, the nurse observes the newborn's color is pink but slightly mottled, arms and legs are limp and extended, hiccups are present, and heart rate is regular and rapid. The nurse should recognize these behaviors as manifestations of: a. stress. b. subtle seizures. c. preterm behavior. d. onset of respiratory distress.

A Color pink but slightly mottled, arms and legs limp and extended, hiccups, respiratory pauses and gasping, and an irregular, rapid heart rate are signs of stress or fatigue in a newborn. Neonatal seizures usually have some type of repetitive movement from twitching to rhythmic jerking movements. The behavior of a preterm newborn may be inactive and listless. Respiratory distress is exhibited by retractions and nasal flaring.

The nurse is planning care for a newborn receiving IV calcium gluconate for treatment of hypocalcemia. Which intervention is the most appropriate during the acute phase? a. Allow newborn to sleep with pacifier to decrease stimuli. b. Keep newborn awake to monitor central nervous system changes. c. Encourage parents to hold and feed newborn to facilitate attachment during illness. d. Awaken newborn periodically to assess level of consciousness.

A For newborns with hypocalcemia, the nurse should manipulate the environment to reduce stimuli that might precipitate a seizure or tremors. A quiet, nonstimulating environment should be maintained for the newborn until calcium levels are normalized. Care should be provided without sudden jarring. Parents can be involved in observations and care when the child is awake.

A preterm newborn requires oxygen and mechanical ventilation. Which complications should the nurse assess for? a. Bronchopulmonary dysplasia, pneumothorax b. Anemia, necrotizing enterocolitis c. Cerebral palsy, persistent patent ductus d. Congestive heart failure, cerebral edema

A Oxygen therapy, although lifesaving, is not without hazards. The positive pressure created by mechanical ventilation creates an increase in the number of ruptured alveoli and subsequent pneumothorax and bronchopulmonary dysplasia. Anemia, necrotizing enterocolitis, cerebral palsy, persistent patent ductus, congestive heart failure, and cerebral edema are complications not primarily due to oxygen therapy and mechanical ventilation.

A preterm newborn, after spending 8 weeks in the NICU, is being discharged. The parents of the newborn express apprehension and worry that the newborn may still be in danger. How should the nurse interpret these statements? a. Normal b. A reason to postpone discharge c. Suggestive of maladaptation d. Suggestive of inadequate bonding

A Parents become apprehensive and excited as the time for discharge approaches. They have many concerns and insecurities regarding the care of their newborn. A major concern is that they may be unable to recognize signs of illness or distress in their newborn. Preparation for discharge should begin early and include helping the parent acquire the skills necessary for care. Apprehension and worry are normal adaptive responses. The NICU nurses should facilitate discharge by involving parents in care as soon as possible.

The nurse is caring for a preterm newborn who requires mechanical ventilation for the treatment of respiratory distress syndrome. What is the preterm newborn at increased risk of due to the mechanical ventilation? a. Alveolar rupture b. Meconium aspiration c. Transient tachypnea d. Retractions and nasal flaring

A Positive pressure introduced by mechanical apparatus has created an increase in the incidence of ruptured alveoli and subsequent pneumothorax and bronchopulmonary dysplasia. Meconium aspiration is not associated with mechanical ventilation. Tachypnea may be an indication of a pneumothorax, but it would not be transient. Retractions and nasal flaring are indications of the use of accessory muscles when the newborn cannot obtain sufficient oxygen. The use of mechanical ventilation bypasses the newborn's need to use these muscles.

To whom is RhIG (RhoGAM) administered to prevent Rh isoimmunization? a. Rh-negative women who deliver an Rh-positive newborn b. Rh-positive women who deliver an Rh-negative newborn c. Rh-negative newborns whose mothers are Rh positive d. Rh-positive fathers before conception of second newborn when first newborn was Rh positive

A RhIG human gamma globulin concentrate of anti-D is administered to all unsensitized Rh-negative women after delivery or abortion of an Rh-positive newborn or fetus. Administering RhIG to an individual who is Rh positive will result in agglutination of red cells and hemolysis. It will not alter the person's genetic makeup. The anti-D antibody contained in RhIG will have no effect on Rh-negative newborns because the D antibody is not present.

The nurse is caring for a newborn with respiratory distress syndrome. The newborn has an endotracheal tube. Which statement describes nursing considerations related to suctioning? a. Suctioning should not be carried out routinely. b. Newborn should be in Trendelenburg position for suctioning. c. Routine suctioning, usually every 15 minutes, is necessary. d. Frequent suctioning is necessary to maintain patency of bronchi.

A Suctioning is not an innocuous procedure and can cause bronchospasm, bradycardia, hypoxia, and increased ICP. It should never be carried out routinely. The Trendelenburg position should be avoided. This position can contribute to increased ICP and reduced lung capacity from gravity pushing organs against diaphragm. Routine suctioning is avoided because of the potential complications of bronchospasm, bradycardia, hypoxia, and increased ICP.

The nurse is caring for a newborn whose mother is diabetic. Which clinical manifestations should the nurse expect to see? a. Hypoglycemic, large for gestational age b. Hyperglycemic, large for gestational age c. Hypoglycemic, small for gestational age d. Hyperglycemic, small for gestational age

A The clinical manifestations of a newborn born to a mother with diabetes include being large for gestational age, being plump and full-faced, having abundant vernix caseosa, being listless and lethargic, and having hypoglycemia. These manifestations appear a short time after birth. The newborn is hypoglycemic from increased fetal production of insulin and large for gestational age.

Which is the most appropriate nursing action when intermittently gavage-feeding a preterm newborn? a. Allow formula to flow by gravity. b. Insert tube through nares rather than mouth. c. Avoid letting newborn suck on tube. d. Apply steady pressure to syringe to deliver formula to stomach in a timely manner.

A The formula is allowed to flow by gravity. The length of time to complete the feeding will vary. Preferably, the tube is inserted through the mouth. Newborns are obligatory nose breathers, and the presence of the tube in the nose irritates the nasal mucosa. Passage of the tube through the mouth allows the nurse to observe and evaluate the sucking response. The feeding should not be done under pressure. This procedure is not used as a timesaver for the nurse.

Which is the most common cause of anemia in preterm newborns? a. Frequent blood sampling b. Respiratory distress syndrome c. Meconium aspiration syndrome d. Persistent pulmonary hypertension

A The most common cause of anemia in preterm newborns is frequent blood-sample withdrawal and inadequate erythropoiesis in acutely ill newborns. Microsamples should be used for blood tests, and the amount of blood drawn should be monitored. Respiratory distress syndrome, meconium aspiration syndrome, and persistent pulmonary hypertension are not causes of anemia. They may require frequent blood sampling, which will contribute to the problem of decreased erythropoiesis and anemia.

Which is the central factor responsible for respiratory distress syndrome? a. Deficient surfactant production b. Overproduction of surfactant c. Overdeveloped alveoli d. Absence of alveoli

A The successful adaptation to extrauterine breathing requires numerous factors, which most term newborns successfully accomplish. Preterm newborns with respiratory distress are not able to adjust. The most likely central cause is the abnormal development of the surfactant system. The deficient production of surfactant results in unequal inflation of alveoli on inspiration and the collapse of the alveoli on end expiration. The number and state of development of the alveoli are not a central factors in respiratory distress syndrome. The instability of the alveoli related to the lack of surfactant is the causative issue.

The newborn with severe jaundice is at risk for developing: a. encephalopathy. b. bullous impetigo. c. respiratory distress. d. blood incompatibility.

A Unconjugated bilirubin, which can cross the blood-brain barrier, is highly toxic to neurons. A newborn with severe jaundice is at risk for developing kernicterus or bilirubin encephalopathy. Encephalopathy is a highly infectious bacterial infection of the skin. It has no relation to severe jaundice and is the most likely complication of severe jaundice. A blood incompatibility may be the causative factor for the severe jaundice.

A blood sample for measurement of bilirubin is required from a newborn receiving phototherapy. In what environment should this blood sample be drawn? a. While phototherapy lights are turned off b. While newborn remains under phototherapy lights c. When newborn is covered with a blanket d. When newborn has been off phototherapy for 30 to 60 minutes

A When blood is drawn, phototherapy lights are turned off, and the blood is transported in a covered tube to avoid a false reading as a result of bilirubin destruction in the test tube. The lights will cause a degradation of the bilirubin in the sample, resulting in a falsely lowered result. The newborn does not need to be covered with a blanket. The phototherapy lights must be off. There is no reason to delay obtaining the blood sample. It can be drawn as soon as the lights are turned off.

A nurse is assessing a preterm newborn. Which assessment findings are consistent with prematurity? (Select all that apply.) a. Abundant lanugo over the body b. Ear cartilage soft and pliable c. Flexed body posture d. Deep creases on the sole of the foot e. Skin is bright pink, smooth, and shiny.

A, B, E The preterm newborn has fine lanugo hair that is abundant over the body. The ear cartilage is soft and pliable, and the soles and palms have minimal creases, resulting in a smooth appearance. The preterm newborn's skin is bright pink (often translucent, depending on the degree of immaturity), smooth, and shiny, with small blood vessels clearly visible underneath the thin epidermis. In contrast to full-term infants' overall attitude of flexion and continuous activity, preterm infants may be inactive and listless. The extremities maintain an attitude of extension and remain in any position in which they are placed.

A nurse is admitting a preterm newborn to the NICU. Which interventions should the nurse implement to prevent retinopathy? (Select all that apply.) a. Place on pulse oximetry. b. Decrease exposure to bright, direct lighting. c. Place on a cardiac monitor. d. Cover eyes with an eye shield at night. e. Use supplemental oxygen only when needed.

A, B, E To prevent retinopathy, the nurse should provide preventive care by closely monitoring blood oxygen levels, responding promptly to saturation alarms, and preventing fluctuations in blood oxygen levels. Pulse oximetry is recommended to monitor the infant's oxygenation status during resuscitation and to prevent excessive use of oxygen in both term and preterm infants. Decrease exposure to bright, direct lighting; although exposure to bright light has not been proven to contribute to retinopathy of prematurity, such exposure is undesirable from a neurobehavioral developmental perspective. Use supplemental oxygen judiciously and monitor oxygen blood levels carefully; prevent wide fluctuations in oxygen blood levels (hyperoxia and hypoxia). Placing the newborn on a cardiac monitor will not prevent retinopathy. Covering the eyes with eye shields is not a preventive measure for retinopathy.

The nurse needs to obtain blood for ongoing assessment of a high-risk newborn's progress. Which tests should the nurse monitor? (Select all that apply.) a. Blood glucose b. Complete blood count (CBC) c. Calcium d. Serum electrolytes e. Neonatal prothrombin time (PTT)

A, C, D The most common blood tests done on high-risk newborns are blood glucose, bilirubin, calcium, hematocrit, serum electrolytes, and blood gases. Hematocrits rather than CBCs are performed. This will monitor the red cell volume. Neonatal prothrombin time (PTT) is not a test.

A nurse is planning care for a preterm newborn. Which interventions should the nurse implement for skin care? (Select all that apply.) a. Use cleaning agents with neutral pH. b. Rub skin during drying. c. Use adhesive remover solvent when removing tape. d. Avoid removing adhesives for at least 24 hours. e. Consider pectin barriers beneath adhesives.

A, D, E The skin care for a preterm newborn should include use of pH-neutral cleanser or soaps no more than two or three times a week. Adhesives should not be removed for at least 24 hours after application. Pectin barriers should be used beneath adhesives to protect skin. Avoid rubbing skin during bathing or drying. Do not use adhesive remover, solvents, or bonding agents. Adhesive removal can be facilitated using water, mineral oil, or petrolatum.

The nurse is caring for a newborn who was born 24 hours ago to a mother who received no prenatal care. The newborn is a poor feeder but sucks avidly on his hands. Clinical manifestations also include loose stools, tachycardia, fever, projectile vomiting, sneezing, and generalized sweating. Which should the nurse suspect? a. Seizure disorder b. Narcotic withdrawal c. Placental insufficiency d. Meconium aspiration syndrome

B Newborns exposed to drugs in utero usually show no untoward effects until 12 to 24 hours for heroin or much longer for methadone. The newborn usually has nonspecific signs that may coexist with other conditions such as hypocalcemia and hypoglycemia. In addition, these newborns may have loose stools, tachycardia, fever, projectile vomiting, sneezing, and generalized sweating, which is uncommon in newborns. Loose stools, tachycardia, fever, projectile vomiting, sneezing, and generalized sweating are manifestations not descriptive of seizure activity. Placental insufficiency usually results in a child who is small for gestational age. Meconium aspiration syndrome usually has manifestations of respiratory distress.

Which statement best describes the clinical manifestations of the preterm newborn? a. Head is proportionately small in relation to the body. b. Sucking reflex is absent, weak, or ineffectual. c. Thermostability is well established. d. Extremities remain in attitude of flexion.

B Reflex activity is only partially developed. Sucking is absent, weak, or ineffectual. The preterm newborn's head is proportionately larger than the body. Thermoregulation is poorly developed, and the preterm newborn needs a neutral thermal environment to be provided. The preterm newborn may be listless and inactive compared with the overall attitude of flexion and activity of a full-term newborn.

The nurse is preparing a parent of a newborn for home phototherapy. Which statement made by the parent would indicate a need for further teaching? a. "I should change the baby's position many times during the day." b. "I can dress the baby in lightweight clothing while under phototherapy." c. "I should be sure that the baby's eyelids are closed before applying patches." d. "I can take the patches off the baby during feedings and other caregiving activities."

B The baby should be placed nude under the lights. The newborn should be repositioned frequently to expose all body surfaces to the lights. The newborn's eyelids must be closed before the patches are applied because the corneas may become excoriated if in contact with the dressing. The eye patches should be removed during feedings and other caregiving activities so the newborn can have visual and sensory stimulation.

Which intervention should the nurse implement to maintain the skin integrity of the preterm newborn? a. Cleanse skin with a gentle alkaline-based soap and water. b. Cleanse skin with a neutral pH solution only when necessary. c. Thoroughly rinse skin with plain water after bathing in a mild hexachlorophene solution. d. Avoid cleaning skin.

B The preterm newborn should be given baths no more than two or three times per week with a neutral pH solution. The eyes, oral and diaper areas, and pressure points should be cleansed daily. Alkaline-based soaps might destroy the acid mantle of the skin. They should not be used. The increased permeability of the skin facilitates absorption of the chemical ingredients. The newborn's skin must be cleaned to remove stool and urine, which are irritating to the skin.

The nurse is planning care for a low birth weight newborn. Which is an appropriate nursing intervention to promote adequate oxygenation? a. Place in Trendelenburg position periodically. b. Suction at least every 2 to 3 hours. c. Maintain neutral thermal environment. d. Hyperextend neck with nose pointing to ceiling.

C A neutral thermal environment is one that permits the newborn to maintain a normal core temperature with minimal oxygen consumption and caloric expenditure. The Trendelenburg position should be avoided. This position can contribute to increased intracranial pressure (ICP) and reduced lung capacity from gravity pushing organs against diaphragm. Suctioning should be done only as necessary. Routine suctioning may cause bronchospasm, bradycardia due to vagal nerve stimulation, hypoxia, and increased ICP. Neck hyperextension is avoided because it reduces diameter of trachea.

A nurse is assessing for jaundice in a dark-skinned newborn. Where is the best place to assess for jaundice in this newborn? a. Buttocks b. Tip of nose and sclera c. Sclera, conjunctiva, and oral mucosa d. Palms of hands and soles of feet

C Assessing for jaundice is part of the routine physical assessment in newborns. In dark-skinned newborns, the sclera, conjunctiva, and oral mucosa are the best place to observe jaundice because of the lack of skin pigmentation in these areas. The skin pigmentation in the buttocks, tip of nose and sclera, and palms of hands and soles of feet can mask the appearance of jaundice.

When should the nurse expect breastfeeding-associated jaundice to first appear in a normal newborn? a. 0 to 12 hours b. 12 to 24 hours c. 2 to 4 days d. 4 to 5 days

C Breastfeeding-associated jaundice is caused by decreased milk intake related to decreased caloric and fluid intake by the newborn before the mother's milk is well established. Fasting is associated with decreased hepatic clearance of bilirubin; 0 to 24 hours is too soon. Jaundice within the first 24 hours is associated with hemolytic disease of the newborn; 4 to 5 days is too late. Jaundice at this time may be due to breast milk jaundice.

What is an early clinical manifestation of bilirubin encephalopathy in the newborn? a. Cognitive impairment b. Absence of stooling c. Lethargy or irritability d. Increased or decreased temperature

C Clinical manifestations of bilirubin encephalopathy are those of nervous system depression or excitation. Prodromal symptoms consist of decreased activity, lethargy, irritability, hypotonia, and seizures. Newborns who survive may have evidence of cognitive impairment. Absence of stooling and increased/decreased temperature are not manifestations of bilirubin encephalopathy.

A newborn is diagnosed with retinopathy of prematurity. What should the nurse know about this diagnosis? a. Blindness cannot be prevented. b. No treatment is currently available. c. Cryotherapy and laser therapy are effective treatments. d. Long-term administration of oxygen will be necessary.

C Cryotherapy and laser photocoagulation therapy can be used to minimize the vascular proliferation process that causes the retinal damage. Blindness can be prevented with early recognition and treatment. Cryotherapy and laser therapy can be used to stop the process. Surgical intervention can be used to repair a detached retina if necessary. Long-term administration of oxygen is one of the causes. Oxygen should be used judiciously.

The nurse is caring for a very low birth weight (VLBW) newborn with a peripheral intravenous infusion. Which statement describes nursing considerations regarding infiltration? a. Infiltration occurs infrequently because VLBW newborns are inactive. b. Continuous infusion pumps stop automatically when infiltration occurs. c. Hypertonic solutions can cause severe tissue damage if infiltration occurs. d. Infusion site should be checked for infiltration at least once per 8-hour shift.

C Hypertonic fluids can damage cells if the fluid leaks from the vein. Careful monitoring is required to prevent severe tissue damage. Infiltrations occur for many reasons, not only activity. The vein, catheter, and fluid used all contribute to the possibility of infiltration. The continuous infusion pump may alarm when the pressure increases, but this does not alert the nurse to all infiltrations. Infusion rates and sites should be checked hourly to prevent tissue damage from extravasations, fluid overload, and dehydration.

A preterm newborn has been receiving orogastric feedings of breast milk. The nurse initiates nipple feedings, but the newborn tires easily and has weak sucking and swallowing reflexes. What is the most appropriate nursing intervention? a. Encourage mother to breastfeed. b. Try nipple-feeding preterm newborn formula. c. Resume orogastric feedings of breast milk. d. Resume orogastric feedings of formula.

C If a preterm newborn tires easily or has weak sucking when nipple feedings are initiated, the nurse should resume orogastric feedings with the milk of mother's choice. When nipple feeding is unsuccessful, it is unlikely that the newborn will be able to breastfeed. Breast milk should be continued as long as the mother desires.

The nurse is caring for a newborn with hyperbilirubinemia who is receiving phototherapy. Which is an appropriate nursing intervention for this newborn? a. Apply lotion as prescribed to moisturize skin. b. Maintain nothing-by-mouth (NPO) status to prevent nausea and vomiting. c. Monitor temperature to prevent hypothermia or hyperthermia. d. Keep eye patches on for at least 8 to 12 of every 24 hours.

C Newborns who are receiving phototherapy are at risk for thermoregulation issues. The nurse must monitor the newborn's temperature closely to rapidly detect either hypothermia or hyperthermia. Lotions are not used. They may predispose the newborn to increased tanning or "frying" effect. Newborns receiving phototherapy require additional fluid to compensate for increased fluid losses caused by the lights. The eye patches must be in place whenever the child is under the phototherapy lights.

Physiologic jaundice in a newborn can be caused by: a. fetal-maternal blood incompatibility. b. destruction of red blood cells as a result of antibody reaction. c. liver's inability to bind bilirubin adequately for excretion. d. immature kidneys' inability to hydrolyze and excrete bilirubin.

C Physiologic jaundice is caused by the immature hepatic function of the newborn's liver coupled with the increased load from red blood cell hemolysis. The excess bilirubin from the destroyed red blood cells cannot be excreted from the body. The fetal-maternal blood incompatibility and the associated red cell destruction by antibodies are the causes of hemolytic disease of the newborn. The kidneys are not involved in the excretion of bilirubin.

When is the best time for the neonatal intensive care unit (NICU) nurse to initiate an individualized stimulation program for the preterm newborn? a. As soon as possible after newborn is born b. As soon as parent is available to provide stimulation c. When newborn is over 38 weeks of gestation d. When developmental organization and stability are sufficient

D Newborn stimulation is essential for growth and development. The appropriate time for the introduction of an individualized program is when developmental organization and stability are achieved at approximately 34 and 36 weeks of gestation. The newborn needs to be developmentally ready for a stimulation program. The newborn must be assessed to determine the readiness and appropriateness of the stimulation program. The program should be designed and implemented by the nursing staff. The family can be involved, as the nurses help teach the parents to be responsive to the child's cues, but the stimulation should not depend on the family's availability. An individualized stimulation program should be started when the child is developmentally ready.

Which is characteristic of newborns whose mothers smoked during pregnancy? a. Large for gestational age b. Preterm, but size appropriate for gestational age c. Growth retardation in weight only d. Growth retardation in weight, length, and head circumference

D Newborns born to mothers who smoke had growth failure in weight, length, and chest circumference when compared with newborns of mothers who did not smoke. A dose-effect relation exists. Newborns have significant growth failure, which is related to the number of cigarettes smoked.

The mother of a preterm newborn asks the nurse when she can start breastfeeding. The nurse should explain that breastfeeding can be initiated when her newborn: a. achieves a weight of at least 3 pounds. b. indicates an interest in breastfeeding. c. does not require supplemental oxygen. d. has adequate sucking and swallowing reflexes.

D Research supports that human milk is the best source of nutrition for term and preterm newborns. Preterm newborns should be breastfed as soon as they have adequate sucking and swallowing reflexes and no other complications such as respiratory complications or concurrent illnesses. Weight is not an issue. Interest in breastfeeding can be evaluated by having nonnutritive sucking at the breast during skin-to-skin kangaroo care so the mother and child may become accustomed to each other. Supplemental oxygen can be provided during breastfeeding by using a nasal cannula.

Which should the nurse anticipate in the newborn whose mother used cocaine during pregnancy? a. Seizures b. Hyperglycemia c. Cardiac and respiratory problems d. Neurobehavioral depression or excitability

D The nurse should anticipate neurobehavioral depression or excitability and implement care directed at the newborn's manifestations. Few or no neurologic sequelae appear in newborns born to mothers who use cocaine during pregnancy. The newborn is usually a poor feeder, so hypoglycemia would be a more likely occurrence. Cardiac and respiratory problems are usually not evident in these newborns.


Ensembles d'études connexes

Female & Male Reproductive problems DYNAMIC ATI

View Set

Deferred Revenue & Expenses and Accrued Revenues & Expenses

View Set

Foundations for Living B 2018 : 2. CHRISTIAN EDUCATION

View Set

Chap. 21 & 22: Solid & Hazardous Waste

View Set